Themenbereiche Themenbereiche Profile Hilfe/Anleitungen Help    
Recent Posts Last 1|3|7 Days Suche Suche Tree Tree View  

Linearkombination

ZahlReich - Mathematik Hausaufgabenhilfe » ---- Archiv: Klassen 12/13 » Analytische Geometrie » Vektorrechnung » Archiviert bis 24. September 2002 Archiviert bis Seite 2 » Linearkombination « Zurück Vor »

Autor Beitrag
Seitenanfangvoriger Beitragnächster BeitragSeitenende Link zu diesem Beitrag

Christian Schmidt (christian_s)
Suche alle Beiträge dieser Person in dieser Hauptrubrik
Erfahrenes Mitglied
Benutzername: christian_s

Nummer des Beitrags: 469
Registriert: 02-2002
Veröffentlicht am Donnerstag, den 12. September, 2002 - 18:21:   Beitrag drucken

Stellen sie den Vektor x mithilfe einer Linearkombination dar, die möglichst wenig Vektoren benötigt; a,b,c,d sind reelle Zahlen.

x=a*(1;2;3)+b*(4;-5;-1)+c*(1;0;1)+d*(14;-11;3)

Irgendwie versteh ich hier die Aufgabenstellung nicht so richtig.

MfG
C. Schmidt
Seitenanfangvoriger Beitragnächster BeitragSeitenende Link zu diesem Beitrag

mythos2002 (mythos2002)
Suche alle Beiträge dieser Person in dieser Hauptrubrik
Mitglied
Benutzername: mythos2002

Nummer des Beitrags: 17
Registriert: 03-2002
Veröffentlicht am Donnerstag, den 12. September, 2002 - 20:07:   Beitrag drucken

Hi,

dies ist eine etwas ungewöhnliche, aber durchaus hübsche Aufgabe!

Die Aufgabenstellung ist, dass der Vektor x, weil selbst dreidimensional, als LK (Linearkombination) nur dreier statt von vier Vektoren dargestellt werden soll.

Der Vektor x muss ja, wie die vier Vektoren, aus welchen er als Linearkombination resultiert, ebenfalls aus R3 stammen (also dreidimensional sein). Diese vier Vektoren sind aber selbst schon linear abhängig (weil sie n+1 Vektoren aus Rn sind).

Eine Basis aus R3 besteht nämlich bereits aus drei lin. unabh. Vektoren, jeder weitere Vektor der gleichen Dimension ist schon eine LK dieser Basis.

Wir stellen nun (o.B.d.A.) den Vektor (14;-11;3) als LK der anderen drei dar:

1. ..... 14 = a + 4b + c
2. ..... -11 = 2a - 5b
3. ..... 3 = 3a - b + c
--------------------------
Die Gleichung 3. - 1. ist (hier zufällig) mit 2. identisch! Daher kann z.B. c beliebig gesetzt werden, sei c = 0;

daraus: .. 13a = 26 --> a = 2, b = 3

Der Vektor (14;-11;3) wird nun durch 2*(1;2;3) + 3*(4;-5;-1) ersetzt und die gleichen Vektoren zusammengefaßt:

x = a*(1;2;3) + b*(4;-5;-1) + c*(1;0;1) + d*[2*(1;2;3) + 3*(4;-5;-1)]

x = a*(1;2;3) + b*(4;-5;-1) + c*(1;0;1) + 2d*(1;2;3) + 3d*(4;-5;-1)

x = (a + 2d)*(1;2;3) + (b + 3d)*(4;-5;-1) + c*(1;0;1)

Somit ist der Vektor x als LK der drei Vektoren (1;2;3), (4;-5;-1) und (1;0;1) dargestellt!

Gr
mythos

[o.B.d.A = ohne Beschränk. d. Allgemeinh.]
Seitenanfangvoriger Beitragnächster BeitragSeitenende Link zu diesem Beitrag

Christian Schmidt (christian_s)
Suche alle Beiträge dieser Person in dieser Hauptrubrik
Erfahrenes Mitglied
Benutzername: christian_s

Nummer des Beitrags: 474
Registriert: 02-2002
Veröffentlicht am Donnerstag, den 12. September, 2002 - 20:13:   Beitrag drucken

Vielen Dank für die Antwort.

MfG
C. Schmidt
Seitenanfangvoriger Beitragnächster BeitragSeitenende Link zu diesem Beitrag

mythos2002 (mythos2002)
Suche alle Beiträge dieser Person in dieser Hauptrubrik
Mitglied
Benutzername: mythos2002

Nummer des Beitrags: 18
Registriert: 03-2002
Veröffentlicht am Donnerstag, den 12. September, 2002 - 20:24:   Beitrag drucken

Bemerkung:

Die in dem Gleichungssystem

1. ..... 14 = a + 4b + c
2. ..... -11 = 2a - 5b
3. ..... 3 = 3a - b + c
------------------------------

verwendeten Variablen a, b, c sind nicht identisch mit denen in der Angabe aufscheinenden reellen Zahlen a, b, c, ...!

Um Missinterpretation zu vermeiden, sollte man besser zur Unterscheidung diese anders bezeichnen, etwa

1. ..... 14 = a' + 4b' + c'
2. ..... -11 = 2a' - 5b'
3. ..... 3 = 3a' - b' + c'
--------------------------------
Die Gleichung 3. - 1. ist (hier zufällig) mit 2. identisch! Daher kann z.B. c' beliebig gesetzt werden, sei c' = 0;

daraus: .. 13a' = 26 --> a' = 2, b' = 3
...
usw.

Sorry!

Gr
mYthos
Seitenanfangvoriger Beitragnächster BeitragSeitenende Link zu diesem Beitrag

clara
Suche alle Beiträge dieser Person in dieser Hauptrubrik
Unregistrierter Gast
Veröffentlicht am Freitag, den 13. September, 2002 - 11:06:   Beitrag drucken

Hi,
die drei verbleibenden Vektoren sind aber nicht linear unabhängig, denn es ist z.B.:
(-2/5)*(4,-5,-1)+(13/5)*(1,0,1)=(1,2,3),
d.h. den Vektor x kann man bereits als Linearkombination von nur zwei Vektoren darstellen.
gruß clara
Seitenanfangvoriger Beitragnächster BeitragSeitenende Link zu diesem Beitrag

mythos2002 (mythos2002)
Suche alle Beiträge dieser Person in dieser Hauptrubrik
Mitglied
Benutzername: mythos2002

Nummer des Beitrags: 19
Registriert: 03-2002
Veröffentlicht am Freitag, den 13. September, 2002 - 15:01:   Beitrag drucken

@clara

Sapperlot! Da hast Du recht, das habe ich zum Schluß nicht überprüft ....

Nun, der Rest ist dennoch nicht mehr schwer, man setzt einfach noch in das Ergebnis statt des Vektors (1;2;3) die von Dir angegebene Linearkombination ein!

Danke für den Hinweis!

Gr
mYthos

Beitrag verfassen
Das Senden ist in diesem Themengebiet nicht unterstützt. Kontaktieren Sie den Diskussions-Moderator für weitere Informationen.

ad

Administration Administration Abmelden Abmelden   Previous Page Previous Page Next Page Next Page